Prove the map has a fixed point











up vote
18
down vote

favorite
11












Assume $K$ is a compact metric space with metric $rho$ and $A$ is a map from $K$ to $K$ such that $rho (Ax,Ay) < rho(x,y)$ for $xneq y$. Prove A have a unique fixed point in $K$.



The uniqueness is easy. My problem is to show that there a exist fixed point. $K$ is compact, so every sequence has convergent subsequence. Construct a sequence ${x_n}$ by $x_{n+1}=Ax_{n}$,${x_n}$ has a convergent subsequence ${ x_{n_k}}$, but how to show there is a fixed point using $rho (Ax,Ay) < rho(x,y)$?










share|cite|improve this question




















  • 1




    (1) I think you need to assume $K$ is complete. (2) You have a convergent subsequence; the only thing you can do now is examine the behavior of its limit ...
    – Neal
    Mar 10 '12 at 13:09








  • 6




    @Neal: A metric space is compact iff it is complete and totally bounded, so completeness comes for free with compactness.
    – Brian M. Scott
    Mar 10 '12 at 13:17










  • Oh, I totally missed "compact" in the question. My bad.
    – Neal
    Mar 11 '12 at 0:24















up vote
18
down vote

favorite
11












Assume $K$ is a compact metric space with metric $rho$ and $A$ is a map from $K$ to $K$ such that $rho (Ax,Ay) < rho(x,y)$ for $xneq y$. Prove A have a unique fixed point in $K$.



The uniqueness is easy. My problem is to show that there a exist fixed point. $K$ is compact, so every sequence has convergent subsequence. Construct a sequence ${x_n}$ by $x_{n+1}=Ax_{n}$,${x_n}$ has a convergent subsequence ${ x_{n_k}}$, but how to show there is a fixed point using $rho (Ax,Ay) < rho(x,y)$?










share|cite|improve this question




















  • 1




    (1) I think you need to assume $K$ is complete. (2) You have a convergent subsequence; the only thing you can do now is examine the behavior of its limit ...
    – Neal
    Mar 10 '12 at 13:09








  • 6




    @Neal: A metric space is compact iff it is complete and totally bounded, so completeness comes for free with compactness.
    – Brian M. Scott
    Mar 10 '12 at 13:17










  • Oh, I totally missed "compact" in the question. My bad.
    – Neal
    Mar 11 '12 at 0:24













up vote
18
down vote

favorite
11









up vote
18
down vote

favorite
11






11





Assume $K$ is a compact metric space with metric $rho$ and $A$ is a map from $K$ to $K$ such that $rho (Ax,Ay) < rho(x,y)$ for $xneq y$. Prove A have a unique fixed point in $K$.



The uniqueness is easy. My problem is to show that there a exist fixed point. $K$ is compact, so every sequence has convergent subsequence. Construct a sequence ${x_n}$ by $x_{n+1}=Ax_{n}$,${x_n}$ has a convergent subsequence ${ x_{n_k}}$, but how to show there is a fixed point using $rho (Ax,Ay) < rho(x,y)$?










share|cite|improve this question















Assume $K$ is a compact metric space with metric $rho$ and $A$ is a map from $K$ to $K$ such that $rho (Ax,Ay) < rho(x,y)$ for $xneq y$. Prove A have a unique fixed point in $K$.



The uniqueness is easy. My problem is to show that there a exist fixed point. $K$ is compact, so every sequence has convergent subsequence. Construct a sequence ${x_n}$ by $x_{n+1}=Ax_{n}$,${x_n}$ has a convergent subsequence ${ x_{n_k}}$, but how to show there is a fixed point using $rho (Ax,Ay) < rho(x,y)$?







metric-spaces compactness fixed-point-theorems






share|cite|improve this question















share|cite|improve this question













share|cite|improve this question




share|cite|improve this question








edited Sep 29 '15 at 6:37









Martin Sleziak

44.4k7115268




44.4k7115268










asked Mar 10 '12 at 13:00









Jiangnan Yu

493415




493415








  • 1




    (1) I think you need to assume $K$ is complete. (2) You have a convergent subsequence; the only thing you can do now is examine the behavior of its limit ...
    – Neal
    Mar 10 '12 at 13:09








  • 6




    @Neal: A metric space is compact iff it is complete and totally bounded, so completeness comes for free with compactness.
    – Brian M. Scott
    Mar 10 '12 at 13:17










  • Oh, I totally missed "compact" in the question. My bad.
    – Neal
    Mar 11 '12 at 0:24














  • 1




    (1) I think you need to assume $K$ is complete. (2) You have a convergent subsequence; the only thing you can do now is examine the behavior of its limit ...
    – Neal
    Mar 10 '12 at 13:09








  • 6




    @Neal: A metric space is compact iff it is complete and totally bounded, so completeness comes for free with compactness.
    – Brian M. Scott
    Mar 10 '12 at 13:17










  • Oh, I totally missed "compact" in the question. My bad.
    – Neal
    Mar 11 '12 at 0:24








1




1




(1) I think you need to assume $K$ is complete. (2) You have a convergent subsequence; the only thing you can do now is examine the behavior of its limit ...
– Neal
Mar 10 '12 at 13:09






(1) I think you need to assume $K$ is complete. (2) You have a convergent subsequence; the only thing you can do now is examine the behavior of its limit ...
– Neal
Mar 10 '12 at 13:09






6




6




@Neal: A metric space is compact iff it is complete and totally bounded, so completeness comes for free with compactness.
– Brian M. Scott
Mar 10 '12 at 13:17




@Neal: A metric space is compact iff it is complete and totally bounded, so completeness comes for free with compactness.
– Brian M. Scott
Mar 10 '12 at 13:17












Oh, I totally missed "compact" in the question. My bad.
– Neal
Mar 11 '12 at 0:24




Oh, I totally missed "compact" in the question. My bad.
– Neal
Mar 11 '12 at 0:24










2 Answers
2






active

oldest

votes

















up vote
21
down vote



accepted










Define $f(x):=rho(x,A(x))$; it's a continuous map. (Note $$rho(x,Ax)lerho(x,y)+rho(y,Ay)+rho(Ay,Ax)quadforall x, yin K$$ or $$rho(x,Ax)-rho(y,Ay)lerho(x,y)+rho(Ax,Ay).$$ Reversing the roles of $x,y$ to get $$left|rho(x,Ax)-rho(y,Ay)right|lerho(x,y)+rho(Ax,Ay)<2delta quad text{ whenever }rho(x,y)<delta.$$ That is, $f$ is actually uniformly continuous.)



Let $alpha:=inf_{xin K}f(x)$, then we can find $x_0in K$ such that $alpha=f(x_0)$, since $K$ is compact. If $alpha>0$, then $x_0neq Ax_0$ and $rho(A(Ax_0),Ax_0)<rho(Ax_0,x_0)=alpha$, which is a contradiction. So $alpha=0$ and $x_0$ is a fixed point. The assumption on $A$ makes it unique.





Note that completeness wouldn't be enough in this case, for example consider $mathbb R$ with the usual metric, and $A(x):=sqrt{x^2+1}$. It's the major difference between $rho(Ax,Ay)<rho(x,y)$ for $xneq y$ and the existence of $0<c<1$ such that for all $x,y,$: $rho(Ax,Ay)leq crho(x,y)$.






share|cite|improve this answer























  • Nice proof!Thank you! :))
    – Jiangnan Yu
    Mar 10 '12 at 13:53










  • How do we show that f(x):=ρ(x,A(x)) is indeed continuous?
    – Jacques
    Apr 2 '12 at 9:22






  • 4




    @Jacques: $delta: x mapsto (x,x)$ is continuous, $A$ is continuous, so $g:(x,y) mapsto (x,A(y))$ is continuous, and $d:(x,y) mapsto d(x,y)$ is continuous, so $f(x) = (dcirc g circ delta)(x)$ is a composition of continuous maps, hence it is continuous. Alternatively, use the triangle inequality and the reverse triangle inequality a few times.
    – t.b.
    Apr 2 '12 at 9:52












  • Can someone clarify about uniqueness?
    – Niebla
    Nov 9 '15 at 2:57






  • 2




    @Niebla In general if we have $rho(A(x), A(y))<rho(x,y)$ - note that the inequality is strict - $A$ can only have one fixed point. Let $a, b$ be two fixed points, then $rho(A(a), B(b))<rho(a, b)$, which is a contradiction since both sides of this strict inequality are equal.
    – Jack M
    Dec 27 '15 at 16:53


















up vote
2
down vote













I don't have enough reputation to post a comment to reply to @андрэ 's question regarding where in the proof it is used that $f$ is a continuous function, so I'll post my answer here:



Since we are told that $K$ is a compact set. $f:Krightarrow K$ being continuous implies that the $mathrm{im}(f) = f(K)$ is also a compact set. We also know that compact sets are closed and bounded, which implies the existence of $inf_{xin K} f(x)$.



If it is possible to show that $f(K) subseteq K$ is a closed set, then it is necessarily compact as well:
A subset of a compact set is compact?
However, I am not aware of how you would do this in this case without relying on continuity of $f$.






share|cite|improve this answer





















    Your Answer





    StackExchange.ifUsing("editor", function () {
    return StackExchange.using("mathjaxEditing", function () {
    StackExchange.MarkdownEditor.creationCallbacks.add(function (editor, postfix) {
    StackExchange.mathjaxEditing.prepareWmdForMathJax(editor, postfix, [["$", "$"], ["\\(","\\)"]]);
    });
    });
    }, "mathjax-editing");

    StackExchange.ready(function() {
    var channelOptions = {
    tags: "".split(" "),
    id: "69"
    };
    initTagRenderer("".split(" "), "".split(" "), channelOptions);

    StackExchange.using("externalEditor", function() {
    // Have to fire editor after snippets, if snippets enabled
    if (StackExchange.settings.snippets.snippetsEnabled) {
    StackExchange.using("snippets", function() {
    createEditor();
    });
    }
    else {
    createEditor();
    }
    });

    function createEditor() {
    StackExchange.prepareEditor({
    heartbeatType: 'answer',
    convertImagesToLinks: true,
    noModals: true,
    showLowRepImageUploadWarning: true,
    reputationToPostImages: 10,
    bindNavPrevention: true,
    postfix: "",
    imageUploader: {
    brandingHtml: "Powered by u003ca class="icon-imgur-white" href="https://imgur.com/"u003eu003c/au003e",
    contentPolicyHtml: "User contributions licensed under u003ca href="https://creativecommons.org/licenses/by-sa/3.0/"u003ecc by-sa 3.0 with attribution requiredu003c/au003e u003ca href="https://stackoverflow.com/legal/content-policy"u003e(content policy)u003c/au003e",
    allowUrls: true
    },
    noCode: true, onDemand: true,
    discardSelector: ".discard-answer"
    ,immediatelyShowMarkdownHelp:true
    });


    }
    });














    draft saved

    draft discarded


















    StackExchange.ready(
    function () {
    StackExchange.openid.initPostLogin('.new-post-login', 'https%3a%2f%2fmath.stackexchange.com%2fquestions%2f118536%2fprove-the-map-has-a-fixed-point%23new-answer', 'question_page');
    }
    );

    Post as a guest















    Required, but never shown

























    2 Answers
    2






    active

    oldest

    votes








    2 Answers
    2






    active

    oldest

    votes









    active

    oldest

    votes






    active

    oldest

    votes








    up vote
    21
    down vote



    accepted










    Define $f(x):=rho(x,A(x))$; it's a continuous map. (Note $$rho(x,Ax)lerho(x,y)+rho(y,Ay)+rho(Ay,Ax)quadforall x, yin K$$ or $$rho(x,Ax)-rho(y,Ay)lerho(x,y)+rho(Ax,Ay).$$ Reversing the roles of $x,y$ to get $$left|rho(x,Ax)-rho(y,Ay)right|lerho(x,y)+rho(Ax,Ay)<2delta quad text{ whenever }rho(x,y)<delta.$$ That is, $f$ is actually uniformly continuous.)



    Let $alpha:=inf_{xin K}f(x)$, then we can find $x_0in K$ such that $alpha=f(x_0)$, since $K$ is compact. If $alpha>0$, then $x_0neq Ax_0$ and $rho(A(Ax_0),Ax_0)<rho(Ax_0,x_0)=alpha$, which is a contradiction. So $alpha=0$ and $x_0$ is a fixed point. The assumption on $A$ makes it unique.





    Note that completeness wouldn't be enough in this case, for example consider $mathbb R$ with the usual metric, and $A(x):=sqrt{x^2+1}$. It's the major difference between $rho(Ax,Ay)<rho(x,y)$ for $xneq y$ and the existence of $0<c<1$ such that for all $x,y,$: $rho(Ax,Ay)leq crho(x,y)$.






    share|cite|improve this answer























    • Nice proof!Thank you! :))
      – Jiangnan Yu
      Mar 10 '12 at 13:53










    • How do we show that f(x):=ρ(x,A(x)) is indeed continuous?
      – Jacques
      Apr 2 '12 at 9:22






    • 4




      @Jacques: $delta: x mapsto (x,x)$ is continuous, $A$ is continuous, so $g:(x,y) mapsto (x,A(y))$ is continuous, and $d:(x,y) mapsto d(x,y)$ is continuous, so $f(x) = (dcirc g circ delta)(x)$ is a composition of continuous maps, hence it is continuous. Alternatively, use the triangle inequality and the reverse triangle inequality a few times.
      – t.b.
      Apr 2 '12 at 9:52












    • Can someone clarify about uniqueness?
      – Niebla
      Nov 9 '15 at 2:57






    • 2




      @Niebla In general if we have $rho(A(x), A(y))<rho(x,y)$ - note that the inequality is strict - $A$ can only have one fixed point. Let $a, b$ be two fixed points, then $rho(A(a), B(b))<rho(a, b)$, which is a contradiction since both sides of this strict inequality are equal.
      – Jack M
      Dec 27 '15 at 16:53















    up vote
    21
    down vote



    accepted










    Define $f(x):=rho(x,A(x))$; it's a continuous map. (Note $$rho(x,Ax)lerho(x,y)+rho(y,Ay)+rho(Ay,Ax)quadforall x, yin K$$ or $$rho(x,Ax)-rho(y,Ay)lerho(x,y)+rho(Ax,Ay).$$ Reversing the roles of $x,y$ to get $$left|rho(x,Ax)-rho(y,Ay)right|lerho(x,y)+rho(Ax,Ay)<2delta quad text{ whenever }rho(x,y)<delta.$$ That is, $f$ is actually uniformly continuous.)



    Let $alpha:=inf_{xin K}f(x)$, then we can find $x_0in K$ such that $alpha=f(x_0)$, since $K$ is compact. If $alpha>0$, then $x_0neq Ax_0$ and $rho(A(Ax_0),Ax_0)<rho(Ax_0,x_0)=alpha$, which is a contradiction. So $alpha=0$ and $x_0$ is a fixed point. The assumption on $A$ makes it unique.





    Note that completeness wouldn't be enough in this case, for example consider $mathbb R$ with the usual metric, and $A(x):=sqrt{x^2+1}$. It's the major difference between $rho(Ax,Ay)<rho(x,y)$ for $xneq y$ and the existence of $0<c<1$ such that for all $x,y,$: $rho(Ax,Ay)leq crho(x,y)$.






    share|cite|improve this answer























    • Nice proof!Thank you! :))
      – Jiangnan Yu
      Mar 10 '12 at 13:53










    • How do we show that f(x):=ρ(x,A(x)) is indeed continuous?
      – Jacques
      Apr 2 '12 at 9:22






    • 4




      @Jacques: $delta: x mapsto (x,x)$ is continuous, $A$ is continuous, so $g:(x,y) mapsto (x,A(y))$ is continuous, and $d:(x,y) mapsto d(x,y)$ is continuous, so $f(x) = (dcirc g circ delta)(x)$ is a composition of continuous maps, hence it is continuous. Alternatively, use the triangle inequality and the reverse triangle inequality a few times.
      – t.b.
      Apr 2 '12 at 9:52












    • Can someone clarify about uniqueness?
      – Niebla
      Nov 9 '15 at 2:57






    • 2




      @Niebla In general if we have $rho(A(x), A(y))<rho(x,y)$ - note that the inequality is strict - $A$ can only have one fixed point. Let $a, b$ be two fixed points, then $rho(A(a), B(b))<rho(a, b)$, which is a contradiction since both sides of this strict inequality are equal.
      – Jack M
      Dec 27 '15 at 16:53













    up vote
    21
    down vote



    accepted







    up vote
    21
    down vote



    accepted






    Define $f(x):=rho(x,A(x))$; it's a continuous map. (Note $$rho(x,Ax)lerho(x,y)+rho(y,Ay)+rho(Ay,Ax)quadforall x, yin K$$ or $$rho(x,Ax)-rho(y,Ay)lerho(x,y)+rho(Ax,Ay).$$ Reversing the roles of $x,y$ to get $$left|rho(x,Ax)-rho(y,Ay)right|lerho(x,y)+rho(Ax,Ay)<2delta quad text{ whenever }rho(x,y)<delta.$$ That is, $f$ is actually uniformly continuous.)



    Let $alpha:=inf_{xin K}f(x)$, then we can find $x_0in K$ such that $alpha=f(x_0)$, since $K$ is compact. If $alpha>0$, then $x_0neq Ax_0$ and $rho(A(Ax_0),Ax_0)<rho(Ax_0,x_0)=alpha$, which is a contradiction. So $alpha=0$ and $x_0$ is a fixed point. The assumption on $A$ makes it unique.





    Note that completeness wouldn't be enough in this case, for example consider $mathbb R$ with the usual metric, and $A(x):=sqrt{x^2+1}$. It's the major difference between $rho(Ax,Ay)<rho(x,y)$ for $xneq y$ and the existence of $0<c<1$ such that for all $x,y,$: $rho(Ax,Ay)leq crho(x,y)$.






    share|cite|improve this answer














    Define $f(x):=rho(x,A(x))$; it's a continuous map. (Note $$rho(x,Ax)lerho(x,y)+rho(y,Ay)+rho(Ay,Ax)quadforall x, yin K$$ or $$rho(x,Ax)-rho(y,Ay)lerho(x,y)+rho(Ax,Ay).$$ Reversing the roles of $x,y$ to get $$left|rho(x,Ax)-rho(y,Ay)right|lerho(x,y)+rho(Ax,Ay)<2delta quad text{ whenever }rho(x,y)<delta.$$ That is, $f$ is actually uniformly continuous.)



    Let $alpha:=inf_{xin K}f(x)$, then we can find $x_0in K$ such that $alpha=f(x_0)$, since $K$ is compact. If $alpha>0$, then $x_0neq Ax_0$ and $rho(A(Ax_0),Ax_0)<rho(Ax_0,x_0)=alpha$, which is a contradiction. So $alpha=0$ and $x_0$ is a fixed point. The assumption on $A$ makes it unique.





    Note that completeness wouldn't be enough in this case, for example consider $mathbb R$ with the usual metric, and $A(x):=sqrt{x^2+1}$. It's the major difference between $rho(Ax,Ay)<rho(x,y)$ for $xneq y$ and the existence of $0<c<1$ such that for all $x,y,$: $rho(Ax,Ay)leq crho(x,y)$.







    share|cite|improve this answer














    share|cite|improve this answer



    share|cite|improve this answer








    edited Oct 15 '14 at 17:46









    Fang Jing

    7461418




    7461418










    answered Mar 10 '12 at 13:15









    Davide Giraudo

    124k16150256




    124k16150256












    • Nice proof!Thank you! :))
      – Jiangnan Yu
      Mar 10 '12 at 13:53










    • How do we show that f(x):=ρ(x,A(x)) is indeed continuous?
      – Jacques
      Apr 2 '12 at 9:22






    • 4




      @Jacques: $delta: x mapsto (x,x)$ is continuous, $A$ is continuous, so $g:(x,y) mapsto (x,A(y))$ is continuous, and $d:(x,y) mapsto d(x,y)$ is continuous, so $f(x) = (dcirc g circ delta)(x)$ is a composition of continuous maps, hence it is continuous. Alternatively, use the triangle inequality and the reverse triangle inequality a few times.
      – t.b.
      Apr 2 '12 at 9:52












    • Can someone clarify about uniqueness?
      – Niebla
      Nov 9 '15 at 2:57






    • 2




      @Niebla In general if we have $rho(A(x), A(y))<rho(x,y)$ - note that the inequality is strict - $A$ can only have one fixed point. Let $a, b$ be two fixed points, then $rho(A(a), B(b))<rho(a, b)$, which is a contradiction since both sides of this strict inequality are equal.
      – Jack M
      Dec 27 '15 at 16:53


















    • Nice proof!Thank you! :))
      – Jiangnan Yu
      Mar 10 '12 at 13:53










    • How do we show that f(x):=ρ(x,A(x)) is indeed continuous?
      – Jacques
      Apr 2 '12 at 9:22






    • 4




      @Jacques: $delta: x mapsto (x,x)$ is continuous, $A$ is continuous, so $g:(x,y) mapsto (x,A(y))$ is continuous, and $d:(x,y) mapsto d(x,y)$ is continuous, so $f(x) = (dcirc g circ delta)(x)$ is a composition of continuous maps, hence it is continuous. Alternatively, use the triangle inequality and the reverse triangle inequality a few times.
      – t.b.
      Apr 2 '12 at 9:52












    • Can someone clarify about uniqueness?
      – Niebla
      Nov 9 '15 at 2:57






    • 2




      @Niebla In general if we have $rho(A(x), A(y))<rho(x,y)$ - note that the inequality is strict - $A$ can only have one fixed point. Let $a, b$ be two fixed points, then $rho(A(a), B(b))<rho(a, b)$, which is a contradiction since both sides of this strict inequality are equal.
      – Jack M
      Dec 27 '15 at 16:53
















    Nice proof!Thank you! :))
    – Jiangnan Yu
    Mar 10 '12 at 13:53




    Nice proof!Thank you! :))
    – Jiangnan Yu
    Mar 10 '12 at 13:53












    How do we show that f(x):=ρ(x,A(x)) is indeed continuous?
    – Jacques
    Apr 2 '12 at 9:22




    How do we show that f(x):=ρ(x,A(x)) is indeed continuous?
    – Jacques
    Apr 2 '12 at 9:22




    4




    4




    @Jacques: $delta: x mapsto (x,x)$ is continuous, $A$ is continuous, so $g:(x,y) mapsto (x,A(y))$ is continuous, and $d:(x,y) mapsto d(x,y)$ is continuous, so $f(x) = (dcirc g circ delta)(x)$ is a composition of continuous maps, hence it is continuous. Alternatively, use the triangle inequality and the reverse triangle inequality a few times.
    – t.b.
    Apr 2 '12 at 9:52






    @Jacques: $delta: x mapsto (x,x)$ is continuous, $A$ is continuous, so $g:(x,y) mapsto (x,A(y))$ is continuous, and $d:(x,y) mapsto d(x,y)$ is continuous, so $f(x) = (dcirc g circ delta)(x)$ is a composition of continuous maps, hence it is continuous. Alternatively, use the triangle inequality and the reverse triangle inequality a few times.
    – t.b.
    Apr 2 '12 at 9:52














    Can someone clarify about uniqueness?
    – Niebla
    Nov 9 '15 at 2:57




    Can someone clarify about uniqueness?
    – Niebla
    Nov 9 '15 at 2:57




    2




    2




    @Niebla In general if we have $rho(A(x), A(y))<rho(x,y)$ - note that the inequality is strict - $A$ can only have one fixed point. Let $a, b$ be two fixed points, then $rho(A(a), B(b))<rho(a, b)$, which is a contradiction since both sides of this strict inequality are equal.
    – Jack M
    Dec 27 '15 at 16:53




    @Niebla In general if we have $rho(A(x), A(y))<rho(x,y)$ - note that the inequality is strict - $A$ can only have one fixed point. Let $a, b$ be two fixed points, then $rho(A(a), B(b))<rho(a, b)$, which is a contradiction since both sides of this strict inequality are equal.
    – Jack M
    Dec 27 '15 at 16:53










    up vote
    2
    down vote













    I don't have enough reputation to post a comment to reply to @андрэ 's question regarding where in the proof it is used that $f$ is a continuous function, so I'll post my answer here:



    Since we are told that $K$ is a compact set. $f:Krightarrow K$ being continuous implies that the $mathrm{im}(f) = f(K)$ is also a compact set. We also know that compact sets are closed and bounded, which implies the existence of $inf_{xin K} f(x)$.



    If it is possible to show that $f(K) subseteq K$ is a closed set, then it is necessarily compact as well:
    A subset of a compact set is compact?
    However, I am not aware of how you would do this in this case without relying on continuity of $f$.






    share|cite|improve this answer

























      up vote
      2
      down vote













      I don't have enough reputation to post a comment to reply to @андрэ 's question regarding where in the proof it is used that $f$ is a continuous function, so I'll post my answer here:



      Since we are told that $K$ is a compact set. $f:Krightarrow K$ being continuous implies that the $mathrm{im}(f) = f(K)$ is also a compact set. We also know that compact sets are closed and bounded, which implies the existence of $inf_{xin K} f(x)$.



      If it is possible to show that $f(K) subseteq K$ is a closed set, then it is necessarily compact as well:
      A subset of a compact set is compact?
      However, I am not aware of how you would do this in this case without relying on continuity of $f$.






      share|cite|improve this answer























        up vote
        2
        down vote










        up vote
        2
        down vote









        I don't have enough reputation to post a comment to reply to @андрэ 's question regarding where in the proof it is used that $f$ is a continuous function, so I'll post my answer here:



        Since we are told that $K$ is a compact set. $f:Krightarrow K$ being continuous implies that the $mathrm{im}(f) = f(K)$ is also a compact set. We also know that compact sets are closed and bounded, which implies the existence of $inf_{xin K} f(x)$.



        If it is possible to show that $f(K) subseteq K$ is a closed set, then it is necessarily compact as well:
        A subset of a compact set is compact?
        However, I am not aware of how you would do this in this case without relying on continuity of $f$.






        share|cite|improve this answer












        I don't have enough reputation to post a comment to reply to @андрэ 's question regarding where in the proof it is used that $f$ is a continuous function, so I'll post my answer here:



        Since we are told that $K$ is a compact set. $f:Krightarrow K$ being continuous implies that the $mathrm{im}(f) = f(K)$ is also a compact set. We also know that compact sets are closed and bounded, which implies the existence of $inf_{xin K} f(x)$.



        If it is possible to show that $f(K) subseteq K$ is a closed set, then it is necessarily compact as well:
        A subset of a compact set is compact?
        However, I am not aware of how you would do this in this case without relying on continuity of $f$.







        share|cite|improve this answer












        share|cite|improve this answer



        share|cite|improve this answer










        answered Oct 21 at 8:49









        Matthew O'Brien

        415




        415






























            draft saved

            draft discarded




















































            Thanks for contributing an answer to Mathematics Stack Exchange!


            • Please be sure to answer the question. Provide details and share your research!

            But avoid



            • Asking for help, clarification, or responding to other answers.

            • Making statements based on opinion; back them up with references or personal experience.


            Use MathJax to format equations. MathJax reference.


            To learn more, see our tips on writing great answers.





            Some of your past answers have not been well-received, and you're in danger of being blocked from answering.


            Please pay close attention to the following guidance:


            • Please be sure to answer the question. Provide details and share your research!

            But avoid



            • Asking for help, clarification, or responding to other answers.

            • Making statements based on opinion; back them up with references or personal experience.


            To learn more, see our tips on writing great answers.




            draft saved


            draft discarded














            StackExchange.ready(
            function () {
            StackExchange.openid.initPostLogin('.new-post-login', 'https%3a%2f%2fmath.stackexchange.com%2fquestions%2f118536%2fprove-the-map-has-a-fixed-point%23new-answer', 'question_page');
            }
            );

            Post as a guest















            Required, but never shown





















































            Required, but never shown














            Required, but never shown












            Required, but never shown







            Required, but never shown

































            Required, but never shown














            Required, but never shown












            Required, but never shown







            Required, but never shown







            Popular posts from this blog

            How do I know what Microsoft account the skydrive app is syncing to?

            When does type information flow backwards in C++?

            Grease: Live!